How to find a basis for the kernel and image of a linear transformation matrix











up vote
1
down vote

favorite













Let $$A=
begin{bmatrix}
0 & 0 & 6 & -18 \
0 & 0 & -1 & 3 \
0 & 0 & -2 & 6 \
end{bmatrix}
$$
Find a basis for the kernel and image of the linear transformation $T$ defined by $T(x) =Ax$.






My question is how do you handle the first 2 columns of A which are all zero's? Does that mean the top 2 values of the basis of the kernel are variables? both zero? The methods I know for calculating the basis for the kernel and image of a transformation matrix are not producing the correct answers in WebWork.










share|cite|improve this question
















bumped to the homepage by Community yesterday


This question has answers that may be good or bad; the system has marked it active so that they can be reviewed.



















    up vote
    1
    down vote

    favorite













    Let $$A=
    begin{bmatrix}
    0 & 0 & 6 & -18 \
    0 & 0 & -1 & 3 \
    0 & 0 & -2 & 6 \
    end{bmatrix}
    $$
    Find a basis for the kernel and image of the linear transformation $T$ defined by $T(x) =Ax$.






    My question is how do you handle the first 2 columns of A which are all zero's? Does that mean the top 2 values of the basis of the kernel are variables? both zero? The methods I know for calculating the basis for the kernel and image of a transformation matrix are not producing the correct answers in WebWork.










    share|cite|improve this question
















    bumped to the homepage by Community yesterday


    This question has answers that may be good or bad; the system has marked it active so that they can be reviewed.

















      up vote
      1
      down vote

      favorite









      up vote
      1
      down vote

      favorite












      Let $$A=
      begin{bmatrix}
      0 & 0 & 6 & -18 \
      0 & 0 & -1 & 3 \
      0 & 0 & -2 & 6 \
      end{bmatrix}
      $$
      Find a basis for the kernel and image of the linear transformation $T$ defined by $T(x) =Ax$.






      My question is how do you handle the first 2 columns of A which are all zero's? Does that mean the top 2 values of the basis of the kernel are variables? both zero? The methods I know for calculating the basis for the kernel and image of a transformation matrix are not producing the correct answers in WebWork.










      share|cite|improve this question
















      Let $$A=
      begin{bmatrix}
      0 & 0 & 6 & -18 \
      0 & 0 & -1 & 3 \
      0 & 0 & -2 & 6 \
      end{bmatrix}
      $$
      Find a basis for the kernel and image of the linear transformation $T$ defined by $T(x) =Ax$.






      My question is how do you handle the first 2 columns of A which are all zero's? Does that mean the top 2 values of the basis of the kernel are variables? both zero? The methods I know for calculating the basis for the kernel and image of a transformation matrix are not producing the correct answers in WebWork.







      linear-algebra linear-transformations






      share|cite|improve this question















      share|cite|improve this question













      share|cite|improve this question




      share|cite|improve this question








      edited Oct 9 '17 at 16:21









      Jack

      27k1681196




      27k1681196










      asked Oct 10 '16 at 2:57









      Aaron

      24125




      24125





      bumped to the homepage by Community yesterday


      This question has answers that may be good or bad; the system has marked it active so that they can be reviewed.







      bumped to the homepage by Community yesterday


      This question has answers that may be good or bad; the system has marked it active so that they can be reviewed.
























          3 Answers
          3






          active

          oldest

          votes

















          up vote
          0
          down vote













          By elimination, you will end up with
          $$
          begin{bmatrix}
          0 & 0 & 0 & 0 \
          0 & 0 & -1 & 3 \
          0 & 0 & 0 & 0 \
          end{bmatrix}
          $$
          which gives you the dimension of the image of $T$ and the kernel of $T$.



          Now, you can read from the matrix a basis for the image of $T$. On the other hand
          $$
          -x_3+3x_4=0
          $$
          tells you how to find a basis for the kernel of $T$.






          share|cite|improve this answer




























            up vote
            0
            down vote













            Image is spanned on linearly independent columns of matrix $A$, which follows from definition of matrix of linear transformation. To find it do the row reduction of $A^T$ and its nonzero columns span the image, or equivalently, if you row reduce $A$ then columns with pivots will correspond to columns of $A$ that form image of the transformation.
            Kernel is set of vectors $x$ such that $Ax=0$. To find it's base you need to solve system of equations given by $Ax=0$.






            share|cite|improve this answer




























              up vote
              -1
              down vote













              The kernel basis is:



              $$ left[
              begin{array}{cc}
              0&0&1&1/3\
              end{array}
              right]^T, left[
              begin{array}{cc}
              1&0&0&0\
              end{array}
              right]^T, left[
              begin{array}{cc}
              0&1&0&0\
              end{array}
              right]^T$$



              The image basis is:



              $$left[
              begin{array}{cc}
              6&-1&-2\
              end{array}
              right]^T$$






              share|cite|improve this answer





















                Your Answer





                StackExchange.ifUsing("editor", function () {
                return StackExchange.using("mathjaxEditing", function () {
                StackExchange.MarkdownEditor.creationCallbacks.add(function (editor, postfix) {
                StackExchange.mathjaxEditing.prepareWmdForMathJax(editor, postfix, [["$", "$"], ["\\(","\\)"]]);
                });
                });
                }, "mathjax-editing");

                StackExchange.ready(function() {
                var channelOptions = {
                tags: "".split(" "),
                id: "69"
                };
                initTagRenderer("".split(" "), "".split(" "), channelOptions);

                StackExchange.using("externalEditor", function() {
                // Have to fire editor after snippets, if snippets enabled
                if (StackExchange.settings.snippets.snippetsEnabled) {
                StackExchange.using("snippets", function() {
                createEditor();
                });
                }
                else {
                createEditor();
                }
                });

                function createEditor() {
                StackExchange.prepareEditor({
                heartbeatType: 'answer',
                convertImagesToLinks: true,
                noModals: true,
                showLowRepImageUploadWarning: true,
                reputationToPostImages: 10,
                bindNavPrevention: true,
                postfix: "",
                imageUploader: {
                brandingHtml: "Powered by u003ca class="icon-imgur-white" href="https://imgur.com/"u003eu003c/au003e",
                contentPolicyHtml: "User contributions licensed under u003ca href="https://creativecommons.org/licenses/by-sa/3.0/"u003ecc by-sa 3.0 with attribution requiredu003c/au003e u003ca href="https://stackoverflow.com/legal/content-policy"u003e(content policy)u003c/au003e",
                allowUrls: true
                },
                noCode: true, onDemand: true,
                discardSelector: ".discard-answer"
                ,immediatelyShowMarkdownHelp:true
                });


                }
                });














                 

                draft saved


                draft discarded


















                StackExchange.ready(
                function () {
                StackExchange.openid.initPostLogin('.new-post-login', 'https%3a%2f%2fmath.stackexchange.com%2fquestions%2f1961666%2fhow-to-find-a-basis-for-the-kernel-and-image-of-a-linear-transformation-matrix%23new-answer', 'question_page');
                }
                );

                Post as a guest















                Required, but never shown

























                3 Answers
                3






                active

                oldest

                votes








                3 Answers
                3






                active

                oldest

                votes









                active

                oldest

                votes






                active

                oldest

                votes








                up vote
                0
                down vote













                By elimination, you will end up with
                $$
                begin{bmatrix}
                0 & 0 & 0 & 0 \
                0 & 0 & -1 & 3 \
                0 & 0 & 0 & 0 \
                end{bmatrix}
                $$
                which gives you the dimension of the image of $T$ and the kernel of $T$.



                Now, you can read from the matrix a basis for the image of $T$. On the other hand
                $$
                -x_3+3x_4=0
                $$
                tells you how to find a basis for the kernel of $T$.






                share|cite|improve this answer

























                  up vote
                  0
                  down vote













                  By elimination, you will end up with
                  $$
                  begin{bmatrix}
                  0 & 0 & 0 & 0 \
                  0 & 0 & -1 & 3 \
                  0 & 0 & 0 & 0 \
                  end{bmatrix}
                  $$
                  which gives you the dimension of the image of $T$ and the kernel of $T$.



                  Now, you can read from the matrix a basis for the image of $T$. On the other hand
                  $$
                  -x_3+3x_4=0
                  $$
                  tells you how to find a basis for the kernel of $T$.






                  share|cite|improve this answer























                    up vote
                    0
                    down vote










                    up vote
                    0
                    down vote









                    By elimination, you will end up with
                    $$
                    begin{bmatrix}
                    0 & 0 & 0 & 0 \
                    0 & 0 & -1 & 3 \
                    0 & 0 & 0 & 0 \
                    end{bmatrix}
                    $$
                    which gives you the dimension of the image of $T$ and the kernel of $T$.



                    Now, you can read from the matrix a basis for the image of $T$. On the other hand
                    $$
                    -x_3+3x_4=0
                    $$
                    tells you how to find a basis for the kernel of $T$.






                    share|cite|improve this answer












                    By elimination, you will end up with
                    $$
                    begin{bmatrix}
                    0 & 0 & 0 & 0 \
                    0 & 0 & -1 & 3 \
                    0 & 0 & 0 & 0 \
                    end{bmatrix}
                    $$
                    which gives you the dimension of the image of $T$ and the kernel of $T$.



                    Now, you can read from the matrix a basis for the image of $T$. On the other hand
                    $$
                    -x_3+3x_4=0
                    $$
                    tells you how to find a basis for the kernel of $T$.







                    share|cite|improve this answer












                    share|cite|improve this answer



                    share|cite|improve this answer










                    answered Oct 10 '16 at 3:23









                    Jack

                    27k1681196




                    27k1681196






















                        up vote
                        0
                        down vote













                        Image is spanned on linearly independent columns of matrix $A$, which follows from definition of matrix of linear transformation. To find it do the row reduction of $A^T$ and its nonzero columns span the image, or equivalently, if you row reduce $A$ then columns with pivots will correspond to columns of $A$ that form image of the transformation.
                        Kernel is set of vectors $x$ such that $Ax=0$. To find it's base you need to solve system of equations given by $Ax=0$.






                        share|cite|improve this answer

























                          up vote
                          0
                          down vote













                          Image is spanned on linearly independent columns of matrix $A$, which follows from definition of matrix of linear transformation. To find it do the row reduction of $A^T$ and its nonzero columns span the image, or equivalently, if you row reduce $A$ then columns with pivots will correspond to columns of $A$ that form image of the transformation.
                          Kernel is set of vectors $x$ such that $Ax=0$. To find it's base you need to solve system of equations given by $Ax=0$.






                          share|cite|improve this answer























                            up vote
                            0
                            down vote










                            up vote
                            0
                            down vote









                            Image is spanned on linearly independent columns of matrix $A$, which follows from definition of matrix of linear transformation. To find it do the row reduction of $A^T$ and its nonzero columns span the image, or equivalently, if you row reduce $A$ then columns with pivots will correspond to columns of $A$ that form image of the transformation.
                            Kernel is set of vectors $x$ such that $Ax=0$. To find it's base you need to solve system of equations given by $Ax=0$.






                            share|cite|improve this answer












                            Image is spanned on linearly independent columns of matrix $A$, which follows from definition of matrix of linear transformation. To find it do the row reduction of $A^T$ and its nonzero columns span the image, or equivalently, if you row reduce $A$ then columns with pivots will correspond to columns of $A$ that form image of the transformation.
                            Kernel is set of vectors $x$ such that $Ax=0$. To find it's base you need to solve system of equations given by $Ax=0$.







                            share|cite|improve this answer












                            share|cite|improve this answer



                            share|cite|improve this answer










                            answered Apr 19 '17 at 17:58









                            Tomasz Tarka

                            424211




                            424211






















                                up vote
                                -1
                                down vote













                                The kernel basis is:



                                $$ left[
                                begin{array}{cc}
                                0&0&1&1/3\
                                end{array}
                                right]^T, left[
                                begin{array}{cc}
                                1&0&0&0\
                                end{array}
                                right]^T, left[
                                begin{array}{cc}
                                0&1&0&0\
                                end{array}
                                right]^T$$



                                The image basis is:



                                $$left[
                                begin{array}{cc}
                                6&-1&-2\
                                end{array}
                                right]^T$$






                                share|cite|improve this answer

























                                  up vote
                                  -1
                                  down vote













                                  The kernel basis is:



                                  $$ left[
                                  begin{array}{cc}
                                  0&0&1&1/3\
                                  end{array}
                                  right]^T, left[
                                  begin{array}{cc}
                                  1&0&0&0\
                                  end{array}
                                  right]^T, left[
                                  begin{array}{cc}
                                  0&1&0&0\
                                  end{array}
                                  right]^T$$



                                  The image basis is:



                                  $$left[
                                  begin{array}{cc}
                                  6&-1&-2\
                                  end{array}
                                  right]^T$$






                                  share|cite|improve this answer























                                    up vote
                                    -1
                                    down vote










                                    up vote
                                    -1
                                    down vote









                                    The kernel basis is:



                                    $$ left[
                                    begin{array}{cc}
                                    0&0&1&1/3\
                                    end{array}
                                    right]^T, left[
                                    begin{array}{cc}
                                    1&0&0&0\
                                    end{array}
                                    right]^T, left[
                                    begin{array}{cc}
                                    0&1&0&0\
                                    end{array}
                                    right]^T$$



                                    The image basis is:



                                    $$left[
                                    begin{array}{cc}
                                    6&-1&-2\
                                    end{array}
                                    right]^T$$






                                    share|cite|improve this answer












                                    The kernel basis is:



                                    $$ left[
                                    begin{array}{cc}
                                    0&0&1&1/3\
                                    end{array}
                                    right]^T, left[
                                    begin{array}{cc}
                                    1&0&0&0\
                                    end{array}
                                    right]^T, left[
                                    begin{array}{cc}
                                    0&1&0&0\
                                    end{array}
                                    right]^T$$



                                    The image basis is:



                                    $$left[
                                    begin{array}{cc}
                                    6&-1&-2\
                                    end{array}
                                    right]^T$$







                                    share|cite|improve this answer












                                    share|cite|improve this answer



                                    share|cite|improve this answer










                                    answered Oct 10 '16 at 3:32









                                    Aaron

                                    24125




                                    24125






























                                         

                                        draft saved


                                        draft discarded



















































                                         


                                        draft saved


                                        draft discarded














                                        StackExchange.ready(
                                        function () {
                                        StackExchange.openid.initPostLogin('.new-post-login', 'https%3a%2f%2fmath.stackexchange.com%2fquestions%2f1961666%2fhow-to-find-a-basis-for-the-kernel-and-image-of-a-linear-transformation-matrix%23new-answer', 'question_page');
                                        }
                                        );

                                        Post as a guest















                                        Required, but never shown





















































                                        Required, but never shown














                                        Required, but never shown












                                        Required, but never shown







                                        Required, but never shown

































                                        Required, but never shown














                                        Required, but never shown












                                        Required, but never shown







                                        Required, but never shown







                                        Popular posts from this blog

                                        Can a sorcerer learn a 5th-level spell early by creating spell slots using the Font of Magic feature?

                                        Does disintegrating a polymorphed enemy still kill it after the 2018 errata?

                                        A Topological Invariant for $pi_3(U(n))$